Question and Answers Forum

All Questions      Topic List

None Questions

Previous in All Question      Next in All Question      

Previous in None      Next in None      

Question Number 172323 by Giantyusuf last updated on 25/Jun/22

which number is greater  22^(55)  and 55^(22)  ???

$${which}\:{number}\:{is}\:{greater} \\ $$$$\mathrm{22}^{\mathrm{55}} \:\boldsymbol{{and}}\:\mathrm{55}^{\mathrm{22}} \:??? \\ $$

Answered by nurtani last updated on 25/Jun/22

22^(55)  > 55^(22)

$$\mathrm{22}^{\mathrm{55}} \:>\:\mathrm{55}^{\mathrm{22}} \\ $$

Answered by floor(10²Eta[1]) last updated on 25/Jun/22

2^(55) .11^(55) >5^(22) .11^(22)   2^(55) .11^(33) >5^(22)   clearly true since 11^(33) >5^(33) >5^(22)   ⇒22^(55) >55^(22)

$$\mathrm{2}^{\mathrm{55}} .\mathrm{11}^{\mathrm{55}} >\mathrm{5}^{\mathrm{22}} .\mathrm{11}^{\mathrm{22}} \\ $$$$\mathrm{2}^{\mathrm{55}} .\mathrm{11}^{\mathrm{33}} >\mathrm{5}^{\mathrm{22}} \\ $$$$\mathrm{clearly}\:\mathrm{true}\:\mathrm{since}\:\mathrm{11}^{\mathrm{33}} >\mathrm{5}^{\mathrm{33}} >\mathrm{5}^{\mathrm{22}} \\ $$$$\Rightarrow\mathrm{22}^{\mathrm{55}} >\mathrm{55}^{\mathrm{22}} \\ $$

Answered by Gazella thomsonii last updated on 25/Jun/22

  A=22^(55)   B=55^(22)   lnA=55ln(22)  ln(B)=22ln(55)  ln(22)≈3.091  ln(55)≈4.007  lnA=165.xxx  lnB=88.xxx  ∴ B<A

$$ \\ $$$$\mathrm{A}=\mathrm{22}^{\mathrm{55}} \:\:\mathrm{B}=\mathrm{55}^{\mathrm{22}} \\ $$$$\mathrm{lnA}=\mathrm{55ln}\left(\mathrm{22}\right)\:\:\mathrm{ln}\left(\mathrm{B}\right)=\mathrm{22ln}\left(\mathrm{55}\right) \\ $$$$\mathrm{ln}\left(\mathrm{22}\right)\approx\mathrm{3}.\mathrm{091}\:\:\mathrm{ln}\left(\mathrm{55}\right)\approx\mathrm{4}.\mathrm{007} \\ $$$$\mathrm{lnA}=\mathrm{165}.\mathrm{xxx}\:\:\mathrm{lnB}=\mathrm{88}.\mathrm{xxx} \\ $$$$\therefore\:\mathrm{B}<\mathrm{A} \\ $$

Answered by MJS_new last updated on 25/Jun/22

f(x)=x^(1/x)  has its maximum at x=e ⇒  ∀a,b∣e<a<b: a^(1/a) >b^(1/b)   ⇔  ((ln a)/a)>((ln b)/b)  ⇔  bln a >aln b  ⇔  a^b >b^a     for a ,b ∈N∧a, b>1 the only exceptions are  2^3 <3^2   2^4 =4^2

$${f}\left({x}\right)={x}^{\frac{\mathrm{1}}{{x}}} \:\mathrm{has}\:\mathrm{its}\:\mathrm{maximum}\:\mathrm{at}\:{x}=\mathrm{e}\:\Rightarrow \\ $$$$\forall{a},{b}\mid\mathrm{e}<{a}<{b}:\:{a}^{\frac{\mathrm{1}}{{a}}} >{b}^{\frac{\mathrm{1}}{{b}}} \\ $$$$\Leftrightarrow \\ $$$$\frac{\mathrm{ln}\:{a}}{{a}}>\frac{\mathrm{ln}\:{b}}{{b}} \\ $$$$\Leftrightarrow \\ $$$${b}\mathrm{ln}\:{a}\:>{a}\mathrm{ln}\:{b} \\ $$$$\Leftrightarrow \\ $$$${a}^{{b}} >{b}^{{a}} \\ $$$$ \\ $$$$\mathrm{for}\:{a}\:,{b}\:\in\mathbb{N}\wedge{a},\:{b}>\mathrm{1}\:\mathrm{the}\:\mathrm{only}\:\mathrm{exceptions}\:\mathrm{are} \\ $$$$\mathrm{2}^{\mathrm{3}} <\mathrm{3}^{\mathrm{2}} \\ $$$$\mathrm{2}^{\mathrm{4}} =\mathrm{4}^{\mathrm{2}} \\ $$

Terms of Service

Privacy Policy

Contact: info@tinkutara.com